Geomania.Org Forumları

Geomania Olimpiyat Denemeleri => Geomania Olimpiyat Denemeleri => Konuyu başlatan: AtakanCİCEK - Eylül 05, 2019, 04:56:44 ös

Başlık: I. Aşama Deneme sınavı
Gönderen: AtakanCİCEK - Eylül 05, 2019, 04:56:44 ös
Arkadaşlar hazırlamış olduğum TUBİTAK deneme sınavını paylaşmak istiyorum. PDF deki yazım hatalarını düzelttiği için metonster'e teşekkür ediyorum. Yazım hatası var ise bildirirseniz mutlu olurum.
Sınav süresi $3$ saattir.

Not:$9$  nolu soruda şıkları hatalı vermişim. Uygun bir vakitte düzelteceğim.

https://docdro.id/0onGYkO

Cevap Anahtarı
$1-C , 2-A , 3-C, 4-A , 5-D , 6-A , 7-C , 8-E(20250) , \\ 9-D , 10-C , 11-D , 12-B , 13-B , 14-D , 15-B , 16-E(50) , \\ 17-C , 18-E(10) , 19-E ( 8 ) , 20-D , 21-C , 22-E (2k \text{  formatındaki sayılar}), \\ 23-A, 24 -A , 25-A, 26-B , 27-E(27) , 28 - C , 29-E , 30-B , 31-E(\dfrac{3}{2}) , 32-C$ 
Başlık: Ynt: I. Aşama Deneme sınavı
Gönderen: DrLucky - Mart 02, 2022, 11:38:35 ös
Arkadaşlar hazırlamış olduğum TUBİTAK deneme sınavını paylaşmak istiyorum. PDF deki yazım hatalarını düzelttiği için metonster'e teşekkür ediyorum. Yazım hatası var ise bildirirseniz mutlu olurum.
Sınav süresi $3$ saattir.

Not:$9$  nolu soruda şıkları hatalı vermişim. Uygun bir vakitte düzelteceğim.

https://docdro.id/0onGYkO

Cevap Anahtarı
$1-C , 2-A , 3-C, 4-A , 5-D , 6-A , 7-C , 8-E(20250) , \\ 9-D , 10-C , 11-D , 12-B , 13-B , 14-D , 15-B , 16-E(50) , \\ 17-C , 18-E(10) , 19-E ( 8 ) , 20-D , 21-C , 22-E (2k \text{  formatındaki sayılar}), \\ 23-A, 24 -A , 25-A, 26-B , 27-E(27) , 28 - C , 29-E , 30-B , 31-E(\dfrac{3}{2}) , 32-C$

Çözümleri bulabileceğimiz bir yer var mıdır?

Bir de 9. soruda dış teğet çember uzunluğu derken hangi uzunluktan bahsediyor?

Cevap: Dış teğet çemberlerin uzunluğu derken dış teğet çemberlerin yarıçaplarının uzunluğu demek istemiş. (L. Gökçe)
Başlık: Ynt: I. Aşama Deneme sınavı
Gönderen: AtakanCİCEK - Mart 09, 2022, 05:14:50 ös
Çözümlerin olduğu listeyi malesef bulamadım fakat bu yoruma çözümleri ekleyeceğim. Not: 32 soruyu da eklediğimde çözümlerin pdf halini koyacağım.
$1)$ : https://geomania.org/forum/index.php?topic=6351.0

$2)$ :  Üslü sayıların farkı bulunan diyafont denklemler genellikle üslerin çift olduğunun ispatlanıp iki kare farkı uygulanmasıyla çözülür.
İspatlamak için $8$ modunda inceleyecek olursak
   $p \not = 2$ için $p$ tek sayıdır. O halde $p\equiv 1,3 (mod4)$ olabilir. Buradan $$p^2\equiv 1,9 (mod16)$$yani $$p^2\equiv 1(mod8)$$ olduğu görülebilir.

a) $k=2a$ , $n=2b+1$ olsun.
$5^{2a}\equiv 1(mod8)$ , $3^{2b+1}\equiv 3(mod8)$ olur. Buradan $5^k-3^n\equiv -2 \equiv 6(mod8)$ olarak bulunur. Ancak eşitliğin sağ tarafının modu 1 olduğu için yanlıştır.

b) $k=2a+1$ ,  $n=2b$ olsun. 
$5^{2a+1}\equiv 5(mod8)$  ve $ 3^{2b}\equiv 1(mod8)$ olduğu görülebilir. Buradan ise $5^k-3^n\equiv 4(mod8)$ olur bu da $p=2$ haricinde mümkün değildir.

c) $k=2a+1$ , $n=2b+1$ olsun.
$5^{2a+1}\equiv 5(mod8)$ ve $ 3^{2b+1}\equiv 3(mod8)$ olur.  Buradan $ 5^k-3^n \equiv 2 (mod8)$ elde edilir ki bu mümkün değildir.

d) $k=2a , n=2b $ olsun.
Buradan ise $$5^k-3^n=(5^a-3^b) . (5^a+3^b) = p^2$$ yazılabilir.  Eşitliğin sağ tarafındaki ifadenin çarpanlarına ayrılmış halleri $\{1,p^2\}\{p,p\}\{p^2,1\}$ olabilir. Eşitliğin sol tarafında kalan terimleri eşlemeyi denediğimizde
$5^a-3^b=1$ ve $5^a+3^b=p^2$olduğu görülebilir. $5^a-3^b$ ifadesini sırasıyla 3 ve 5 modlarında incelersek
$2^a\equiv 1(mod3)$ve $-3^b\equiv 1(mod5)$ eşitsizlikleri elde edilir. İlk denkliğe bakıldığında $a$ sayısının çift olduğu görülebilir. İkinci denklik ise $3^b\equiv -1(mod5)$  $b$ nin aldığı her $4$ değerde tekrarladığı göz önüne alınarak $b\equiv 2(mod4)$olduğu elde edilebilir yani çifttir. Ancak $5^a-3^b= (5^{a/2}-3^{b/2}).((5^{a/2}-3^{b/2}))=1$ olduğundan dolayı  $5^{a/2}-3^{b/2}=1$ ve $(5^{a/2}-3^{b/2}=1$ yani bu da $3^{b/2}=0$ olur,  çelişkidir.

O halde $k=2a+1 , n = 2b$ olmalıdır. Bu durum için $p=2$ olduğunu da göz önüne alırsak $5^{2a+1}-3^{2b}=4$ olur. Bu denklemi $b>1$ için $3$ modunda incelersek  $$ 5.25^a -3^{2b}\equiv2 (mod3)$$ olur. Bu da eşitsizliğin sağ tarafı ile çelişir. O halde $b=0$ olmalıdır. $5.25^a-1=4$ buradan ise $a=0$ olur,  $k=1 , n=0 , p=2$ denklemin tek çözümü olur.

$3)$ $x^6+2x^5+2x^4-2x^3-10x^2-10x-3=0$ eşitliğinin $x=-1$ için sağlandığı tahmin edilebilir. Polinom bölmesi yardımıyla $(x+1).(x^5+x^4+x^3-3x^2-7x-3)=0$ olur. $x=-1$ ilk çözümdür.
$x^5+x^4+x^3-3x^2-7x-3= x^4(x+1)+x^3-x -3x^2-6x-3=x^4.(x+1)+(x^2-x).(x+1)-3.(x+1)^2=(x+1).(x^4+x^2-4x-3)$ olduğu elde edilir.  $x\not = -1$ için $x^4+x^2-4x-3=0$ elde edilir. 4. dereceden denklemlerin çözüm yollarından biri olarak  $(x^2+ax+b).(x^2+cx+d) = x^4+x^2-4x-3$ şeklinde polinom eşitliği kullanılabilir. Dağılımı yaparsak  $$ x^4+ (a+c) x^3 + (ac+b+d) x^2+ (ad+bc) x +bd$$ elde edilir.
$$ a+c= 0 , ac+b+d= 1 , ad+bc= -4 , bd=-3$$ eşitlikleri elde edilir. Bu denklemin çözümünü bulmanın yollarından biri $bd=-3$ ifadesinde $b$ ve $d$ ye değerler vererek $a$ ve $c$ nin diğer eşitlikleri sağladığını görmektir.  $b=3, d=-1$ için $ ac+3-1=1$ yani $ac=-1$ ve $a+c=0$ elde edilir. Buradan $a=1$, $c=-1$ veya $a=-1$, $c=1$  olduğu tahmin edilebilir. $ad+bc=-4$ eşitliğini sağlayanı ise $a=1$, $c=-1$ olur.
O halde ifademizin çarpanlarına ayrılmış şekli $x^4+x^2-4x-3=(x^2+x+3).(x^2-x-1)$ olur.   İlk çarpanın $\Delta<0$ olduğundan dolayı reel kökü yoktur. İkinci çarpanın ise kökleri toplamı $1$ dir.  Farklı reel köklerinin toplamı $A=-1+1=0$ buradan $A.(B+1)=0$ elde edilir.

$4)$ Boyaların numaraları $1,2,3,4,5$ olsun. $1$ ile $2$ nin birlikte olduğu ve $3$ ile $4$ ün bulunduğu $2$x$2$ kutular oluşturalım. $1$ ve $2$ numaradan oluşan $2$x$2$ kutuyu tüm köşegen üzerine koyalım. Daha sonra her satır ve sütünda $3$ ve $4$ olmasını sağlamak için $1$ ve $ 2$ numaralarını içeren 1. kutu hariç diğer soluna ve üstüne $3$ ve $4$ içeren kutuları koyalım. Oluşan şekilde kalan tüm boşlukları $5$ ile tamamlarsak $5$ farklı renkle $10$x$10$ luk kutuyu kurallara uygun şekilde boyamış oluruz.
\begin{array}{|c|c|c|c|c|} \hline   1 & 2  & 3 & 4 &  &  &  &  &  &      \\ \hline 2 & 1 &4  & 3 &  &  &  &  &  &      \\ \hline 3 & 4 & 1 & 2  &  &   &  &  &  &       \\ \hline 4 & 3  & 2 & 1 &  &  &  &  &  &      \\ \hline  & & &   & 1 & 2 & 3 & 4 &  &  \\ \hline    &   &  &  & 2 & 1 & 4 & 3 &  &   \\ \hline   &  &  &  & 3 & 4 & 1 & 2 & 3 & 4  \\ \hline &  &  &  & 4 & 3 & 2 & 1 & 4 &  3 \\ \hline &  &  &  &  &  & 3 & 4 & 1 & 2  \\ \hline  &  &  &  &  &  & 4 & 3 & 2 & 1 \\  \hline   \end{array}


$5)$ Değer seçimi yapmadan genel çözümü için https://geomania.org/forum/index.php?topic=6382.0

$6)$ eşitliğin her iki tarafını $a^2$ ile genişletelim.
$$a^2.p=a^4+a^2b^2+a^2c^2+a^2d^2$$ olur.  $a^2b^2=c^2d^2$ yazalım.
$$ a^2. p = a^4+ c^2d^2+a^2c^2+a^2d^2$$ olur. Çarpanlarına ayırırsak
$$a^2.p = (a^2+c^2)(a^2+d^2)$$ $a,b,c,d$ pozitif tamsayılar olduğu için $a^2+c^2>a^2$ ve $a^2+d^2>a^2$ olduğunu biliyoruz. Buradan $a^2$ ifadesi eşitliğin sağ tarafındaki çarpanlardan sadece birinin böleni bile olsa sadeleştirmeden sonra eşitliğin sağ tarafında $1$ den büyük $2$ tane çarpan bulunacağı için $p$ asal sayı olamaz. $0$ çözüm bulunur.
Başlık: Ynt: I. Aşama Deneme sınavı
Gönderen: Metin Can Aydemir - Mart 11, 2022, 10:14:25 ös
Daha önce paylaştığım buradaki (https://geomania.org/forum/index.php?topic=6793.0) "İki Tamkarenin Toplamı" adlı çalışmadan faydalanılarak 6. soruya başka bir çözüm daha getirilebilir.

Lemma:$p$ tek bir asal sayı olmak üzere  $p\equiv 3\pmod{4}$ ise $p$, iki tamkarenin toplamı olarak ifade edilemez. $p\equiv 1 \pmod{4}$ formatında ise tek bir şekilde ifade edilebilir. (Örnek: $5=1^2+2^2$, $13=2^2+3^2$, $101=1^2+10^2$ fakat $x^2+y^2=7$ olacak şekilde $x$ ve $y$ tamsayıları yoktur.)

İspat: Yukarıda belirttiğim konuda ispatı vardır.

Soruya dönersek, ilk olarak $p\neq 2$ olduğu görülebilir çünkü $p=a^2+b^2+c^2+d^2\geq 4$'dür, yani $p$ tektir. $ab=cd$ olduğundan $$p=a^2+b^2+c^2+d^2+2ab-2cd=(a+b)^2+(c-d)^2$$ $$p=a^2+b^2+c^2+d^2-2ab+2cd=(a-b)^2+(c+d)^2$$ olur. Yukarıda verdiğimiz lemmadan dolayı $\left (a+b, |c-d|\right)=\left (|a-b|, c+d\right)$ veya $\left (a+b, |c-d|\right)=\left (c+d,|a-b|\right)$ olmalıdır. $a,b,c,d$ sayıları pozitif olduğundan $a+b\neq |a-b|$ olacaktır. Dolayısıyla $a+b=c+d$ ve $|a-b|=|c-d|$ olur. Genelliği bozmadan $a\geq b$ ve $c\geq d$ dersek, $$2a=(a+b)+|a-b|=(c+d)+|c-d|=2c\implies a=c$$ olur. Buradan $b=d$ sonucu da çıkar. Dolayısıyla, $$a^2+b^2+c^2+d^2=2(a^2+b^2)=p$$ olur. $p$ tek sayı olduğundan bu bir çelişkidir. Çözüm yoktur.
SimplePortal 2.3.3 © 2008-2010, SimplePortal